관리 메뉴


수악중독

2008 AMC12 A #17 본문

AMC12/2008

2008 AMC12 A #17

수악중독 2014. 12. 19. 17:09

Let \(a_1 , \; a_2 , \; \cdots\) be a sequence of integers determined by the rule \(a_n = \dfrac{a_{n-1}}{2}\) if \(a_{n-1}\) is even and \(a_n = 3a_{n-1} +1\) if \(a_{n-1}\) is odd. For how many positive integers \(a_1 \le 2008\) is it true that \(a_1\) is less thatn each of \(a_2 , \; a_3 ,\) and \(a_4\) ?


(A) \(250\)          (B) \(251\)          (C) \(501\)          (D) \(502\)          (E) \(1004\)



Comments